Diễn Đàn MathScopeDiễn Đàn MathScope
  Diễn Đàn MathScope
Ghi Danh Hỏi/Ðáp Thành Viên Social Groups Lịch Ðánh Dấu Ðã Ðọc

Go Back   Diễn Đàn MathScope > Sơ Cấp > Việt Nam và IMO > 2016

News & Announcements

Ngoài một số quy định đã được nêu trong phần Quy định của Ghi Danh , mọi người tranh thủ bỏ ra 5 phút để đọc thêm một số Quy định sau để khỏi bị treo nick ở MathScope nhé !

* Nội quy MathScope.Org

* Một số quy định chung !

* Quy định về việc viết bài trong diễn đàn MathScope

* Nếu bạn muốn gia nhập đội ngũ BQT thì vui lòng tham gia tại đây

* Những câu hỏi thường gặp

* Về việc viết bài trong Box Đại học và Sau đại học


Trả lời Gởi Ðề Tài Mới
 
Ðiều Chỉnh Xếp Bài
Old 24-03-2016, 02:36 PM   #1
Nguyen Van Linh
Moderator
 
Tham gia ngày: Aug 2009
Đến từ: Hà Nội
Bài gởi: 277
Thanks: 69
Thanked 323 Times in 145 Posts
Việt Nam TST 2016 - Đề và lời giải

Kì thi chọn đội tuyển Việt Nam tham dự kì thi toán quốc tế IMO 2016 diễn ra vào hai ngày 24-25/3/2016 tại trường THPT chuyên Hà Nội-Amsterdam.
Mình lập ra topic này mong tập hợp được đề đầy đủ 2 ngày.

Ngày 1.

Bài 1. Tìm $a,n$ nguyên dương với $a>2$ để mỗi ước nguyên tố của $a^n-1$ cũng là ước nguyên tố của $a^{3^{2016}}-1.$

Bài 2. $A$ là tập $2000$ số nguyên phân biệt và $B$ là tập $2016$ số nguyên phân biệt. $K $ là số cặp $(m,n)$ có thứ tự với $m$ thuộc $A$ và $n$ thuộc $B$ mà $|m-n|\leq 1000$. Tìm max $K$?

Bài 3. Cho tam giác $ABC$ nội tiếp đường tròn $(O)$ có $B,C$ cố định, $A$ chuyển động trên cung $BC$ của $ (O)$. Các phân giác $AD,BE,CF$ giao nhau tại $I$. Đường tròn qua $D$ tiếp xúc với $OA$ tại $A$ cắt $(O)$ tại $G$. $GE,GF$ giao $(O)$ lần thứ hai tại $M,N$. $BM$ giao $CN$ tại $H.$
a) Chứng minh rằng $AH$ đi qua một điểm cố định.
b) $BE, CF$ giao $(O)$ lần lượt tại $K,L$. $AH$ giao $KL$ tại $P$. $Q$ là một điểm trên $EF$ sao cho $QP=QI.$ $J$ là điểm nằm trên $(BIC)$ sao cho $IJ\perp IQ$. Chứng minh rằng trung điểm $IJ$ chuyển động trên một đường tròn cố định.

Ngày 2.

Bài 4. Cho tam giác $ABC$ nhọn có $\angle ACB<\angle ABC<\angle ACB+\dfrac{\angle BAC}{2}$. Lấy điểm $D$ thuộc cạnh $BC$ sao cho $\angle ADC=\angle ACB+\dfrac{\angle BAC}{2}$. Tiếp tuyến với đường tròn ngoại tiếp tam giác $ABC$ tại $A$ cắt $BC$ tại $E$. Phân giác $\angle AEB$ cắt $AD$ và cắt $(ADE)$ tại $G$ và $ F$, $DF$ giao $AE$ tại $H.$
a) Chứng minh rằng các đường tròn đường kính $AE,DF,GH$ có một điểm chung.
b) Trên phân giác ngoài $\angle BAC $ và trên tia $AC$ lần lượt lấy các điểm $K$ và $M$ sao cho $KB=KD=KM$, trên phân giác ngoài $\angle BAC$ và trên tia $AB$ lần lượt lấy các điểm $L$ và $N$ sao cho $LC=LD=LN.$ Đường tròn đi qua $M,N$ và trung điểm $I$ của $BC$ cắt $BC$ tại $P$ ($P\neq I$). Chứng minh rằng $BM,CN,AP$ đồng quy.

Bài 5. Cho $a_1,a_2,...,a_{n-1},a_n$ ($n\geq 3$), trong đó mỗi số $a_i $ nhận giá trị $\in \{0;1\}$. Xét $n$ bộ số $S_1=(a_1,a_2,...,a_{n-1},a_n)$, $S_2=(a_2,a_3,...,a_n,a_1)$;...;$S_n=(a_n, a_1, ..., a_{n-2},a_{n-1})$. Với mỗi bộ số $r=(b_1,b_2,...,b_n)$, đặt $\omega(r)=b_1.2^{n-1}+b_2.2^{n-2}+...+b_n.2^0.$ Giả sử các số $\omega(S_1); \omega(S_2);...;\omega(S_n)$ nhận đúng $k $ giá trị phân biệt.
a) Chứng minh rằng $n\vdots k$ và $\omega(S_i)\vdots \dfrac{2^n-1}{2^k-1}$ $\forall i=\overline{1,n}.$
b) Kí hiệu $M$ và $m$ lần lượt là max và min của $\omega(S_1),...,\omega(S_n)$. Chứng minh rằng $M-m\geq \dfrac{(2^n-1)(2^{k-1}-1)}{2^k-1}.$

Bài 6. Cho các số thực phân biệt $\alpha_1,\alpha_2,...,\alpha_{16}$. Với mỗi đa thức hệ số thực $P(x)$; đặt $V(P)=P(\alpha_1)+P(\alpha_2)+...+P(\alpha_{16}).$
Chứng minh rằng tồn tại duy nhất đa thức $Q(x)$ bậc 8 có hệ số $x^8$ bằng $1$ thỏa mãn
i) $V(QP)=0$ với mọi đa thức $P$ có bậc bé hơn $8.$
ii) $Q(x)$ có $8$ nghiệm thực (tính cả bội).
[RIGHT][I][B]Nguồn: MathScope.ORG[/B][/I][/RIGHT]
 

thay đổi nội dung bởi: Nguyen Van Linh, 25-03-2016 lúc 03:01 PM
Nguyen Van Linh is offline   Trả Lời Với Trích Dẫn
The Following 12 Users Say Thank You to Nguyen Van Linh For This Useful Post:
2M (24-03-2016), DaiToan (24-03-2016), hungchng (24-03-2016), huynhcongbang (24-03-2016), lenguyencm (26-03-2016), lovemaths_hn (24-03-2016), Mr_Pi (24-03-2016), namdung (24-03-2016), noh ssiW (24-03-2016), omerta_vn (24-03-2016), pco (24-03-2016), ThangToan (24-03-2016)
Old 24-03-2016, 03:09 PM   #2
Nguyen Van Linh
Moderator
 
Tham gia ngày: Aug 2009
Đến từ: Hà Nội
Bài gởi: 277
Thanks: 69
Thanked 323 Times in 145 Posts
Bài 3.

a) Thực ra câu này cần chứng minh $H$ thuộc phân giác $AD$ và $AD$ luôn đi qua trung điểm cung $BC $ cố định.
Do đường tròn Apollonius trực giao với $(O)$ nên đường tròn đi qua $D$ và tiếp xúc với $(O)$ tại $A$ chính là đường tròn $A$-Apollonius của tam giác $ABC$. Tâm của đường tròn là $X$-giao của tiếp tuyến tại $A$ với $BC$. Ta có $XA^2=XG^2=XB.XC$ nên tứ giác $ABGC$ điều hòa.
Gọi $H'$ là giao của $AD$ với $EF$. Ta có $(AIH'D)=-1$ suy ra $B(AIH'D)=-1.$
Gọi $W$ là giao của $BH'$ với $AC$ suy ra $(AEWC)=-1 \Rightarrow M(AEWC)=-1=(AGBC)$. Do đó $BM$ đi qua $W$ hay $BM$ đi qua $H'$. Tương tự $CN$ đi qua $H'.$ Vậy $H'\equiv H$ và $H\in AD.$

b) Gọi $Z$ là trung điểm cung $BC$ chứa $A$. $AZ$ giao $BC$ tại $V.$
Ta có $A(XLFI)=(ALBS)=C(ALBS)=(AIDS)=B(AIDS)=(AKCS)=A(XK EI).$
Suy ra $AX, KL, EF$ đồng quy tại $U$. Mà $KL$ là trung trực $AI$ nên $UA=UI$. Suy ra $\angle UIA=\angle UAI=1/2A+C=\angle ADX$. Do đó $UI\parallel BC.$
Gọi $Y,Y'$ là giao của đường thẳng qua $I$ vuông góc với $OI$ với $BC$ và $KL$. $YY'$ cắt $AZ$ tại $R$. $YY'$ giao $EF$ tại $Q'$.
Áp dụng định lý con bướm cho $ (O)$, điểm $I$ với hai dây $BK$ và $CL$ đi qua $I$ ta có $IY=IY'$. Mà $Y'A=Y'I,$ $\angle RAI=90^\circ$ nên $Y'R=Y'I=YI.$
Gọi $T$ là giao của $UY'$ với $BC$. Ta có $TY'\parallel VR$ nên $\frac{TV}{TY}=\frac{1}{2}$. $UI$ là đường trung bình của tam giác $TYY'$ nên $UT=UY'$. Áp dụng định lý Menelaus cho tam giác $TYY'$ với 3 điểm $V,U,Q'$ suy ra $Q'Y=1/3Q'Y $ hay $Q'$ là trung điểm $IY'$. Suy ra $PQ'=IQ' $ hay $Q'\equiv Q$. Vậy $O\in IJ$. Gọi $M'$ là trung điểm $IJ$ ta có $\angle OM'S=90^\circ$ nên $M'$ nằm trên $(OS)$ cố định.
[RIGHT][I][B]Nguồn: MathScope.ORG[/B][/I][/RIGHT]
 
Hình Kèm Theo
Kiểu File : png 111.png (55.6 KB, 662 lần tải)

thay đổi nội dung bởi: Nguyen Van Linh, 24-03-2016 lúc 04:24 PM
Nguyen Van Linh is offline   Trả Lời Với Trích Dẫn
The Following 3 Users Say Thank You to Nguyen Van Linh For This Useful Post:
huynhcongbang (24-03-2016), lenguyencm (26-03-2016), omerta_vn (24-03-2016)
Old 24-03-2016, 03:55 PM   #3
Nguyen Van Linh
Moderator
 
Tham gia ngày: Aug 2009
Đến từ: Hà Nội
Bài gởi: 277
Thanks: 69
Thanked 323 Times in 145 Posts
Bài 1 và 2 theo lời của chị Đặng Thu Hương.

Bài 1. Tìm $a,n$ nguyên dương với $a>2$ để mỗi ước nguyên tố của $a^n-1$ cũng là ước nguyên tố của $a^{3^{2016}}-1.$

Bài 2. $A$ là tập $2000$ số nguyên phân biệt và $B$ là tập $2016$ số nguyên phân biệt. $K $ là số cặp $(m,n)$ có thứ tự với $m$ thuộc $A$ và $n$ thuộc $B$ mà $|m-n|\leq 1000$. Tìm max $K$?
[RIGHT][I][B]Nguồn: MathScope.ORG[/B][/I][/RIGHT]
 
Nguyen Van Linh is offline   Trả Lời Với Trích Dẫn
The Following User Says Thank You to Nguyen Van Linh For This Useful Post:
huynhcongbang (24-03-2016)
Old 24-03-2016, 04:35 PM   #4
noh ssiW
+Thành Viên+
 
noh ssiW's Avatar
 
Tham gia ngày: Dec 2011
Bài gởi: 55
Thanks: 1
Thanked 27 Times in 15 Posts
Dự đoán Bài 2: $\max K=2031888$.

[RIGHT][I][B]Nguồn: MathScope.ORG[/B][/I][/RIGHT]
 
__________________
noh ssiW is offline   Trả Lời Với Trích Dẫn
Old 24-03-2016, 04:47 PM   #5
huynhcongbang
Administrator

 
huynhcongbang's Avatar
 
Tham gia ngày: Feb 2009
Đến từ: Ho Chi Minh City
Bài gởi: 2,413
Thanks: 2,165
Thanked 4,188 Times in 1,381 Posts
Gửi tin nhắn qua Yahoo chát tới huynhcongbang
Trích:
Nguyên văn bởi Nguyen Van Linh View Post
Bài 2. $A$ là tập $2000$ số nguyên phân biệt và $B$ là tập $2016$ số nguyên phân biệt. $K $ là số cặp $(m,n)$ có thứ tự với $m$ thuộc $A$ và $n$ thuộc $B$ mà $|m-n|\leq 1000$. Tìm max $K$?
Câu 2 đề có thiếu gì không Linh nhỉ?

Nếu thế thì cứ tham lam, cho $A = \{9, 10, 11, ..., 2008 \}$ và $B = \{1, 2, 3, ..., 2016 \} $ (tức là tập A nằm giữa tập B) thì được $K$ max rồi còn gì?
[RIGHT][I][B]Nguồn: MathScope.ORG[/B][/I][/RIGHT]
 
__________________
Sự im lặng của bầy mèo
huynhcongbang is offline   Trả Lời Với Trích Dẫn
The Following User Says Thank You to huynhcongbang For This Useful Post:
Nguyen Van Linh (25-03-2016)
Old 24-03-2016, 06:20 PM   #6
ThangToan
+Thành Viên+
 
Tham gia ngày: Nov 2010
Đến từ: THPT chuyên Vĩnh Phúc
Bài gởi: 570
Thanks: 24
Thanked 537 Times in 263 Posts
Trích:
Nguyên văn bởi Nguyen Van Linh View Post
Kì thi chọn đội tuyển Việt Nam tham dự kì thi toán quốc tế IMO 2016 diễn ra vào hai ngày 24-25/3/2016 tại trường THPT chuyên Hà Nội-Amsterdam.
Mình lập ra topic này mong tập hợp được đề đầy đủ 2 ngày.

Ngày 1.

Bài 1. Tìm $a,n$ nguyên dương với $a>2$ để mỗi ước nguyên tố của $a^n-1$ cũng là ước nguyên tố của $a^{3^{2016}}-1.$
Bài 1 này sử dụng định lí Zsigmondy Theorem
Định lí. Nếu $a, b$ là hai số nguyên dương $a<b$ và $(a,b)1$ và số $a^n-b^n$ không có dạng $2^6-1$ và $a^2-b^2$, trong đó tổng của $a$ và $b$ không là lũy thừa của 2 thì $a^n-b^n$ có ít nhất một ước nguyên tố $p$ thỏa mãn $p$ không là ước của $a^m-b^m$ với mọi $m<n$.
[RIGHT][I][B]Nguồn: MathScope.ORG[/B][/I][/RIGHT]
 
ThangToan is offline   Trả Lời Với Trích Dẫn
Old 24-03-2016, 09:27 PM   #7
tikita
Administrator

 
Tham gia ngày: Jun 2012
Bài gởi: 157
Thanks: 2
Thanked 84 Times in 53 Posts
Trích:
Nguyên văn bởi Nguyen Van Linh View Post
Kì thi chọn đội tuyển Việt Nam tham dự kì thi toán quốc tế IMO 2016 diễn ra vào hai ngày 24-25/3/2016 tại trường THPT chuyên Hà Nội-Amsterdam.
Mình lập ra topic này mong tập hợp được đề đầy đủ 2 ngày.

Ngày 1.

Bài 2. $A$ là tập $2000$ số nguyên phân biệt và $B$ là tập $2016$ số nguyên phân biệt. $K $ là số cặp $(m,n)$ có thứ tự với $m$ thuộc $A$ và $n$ thuộc $B$ mà $|m-n|\leq 1000$. Tìm max $K$?
Có lẽ đáp án bài này là: 3016944.
Lời giải vắn tắc như sau: Trước hết ta có hai bổ đề sau
Bổ đề 1: Cho $k$ là số nguyên dương thỏa $k\leq 2000-17$ và $a,b$ là hai số nguyên thỏa $|a-b|\geq 2000-k$. Khi đó tổng của số các số nguyên $m\in B$ thỏa $|a-m|\leq 1000$ và số các số $m\in B$ thỏa $|b-m|\leq 1000$ không vượt quá $2017+k$.
Bổ đề 2: Cho $k$ là số nguyên dương sao cho $k\leq 16$ và $a,b$ là hai số nguyên thỏa $|a-b|\geq k$. Khi đó tổng của số các số nguyên $m\in B$ thỏa $|a-m|\leq 1000$ và số các số $m\in B$ thỏa $|b-m|\leq 1000$ không vượt quá $4002$.

Đặt $S(a)$ là số các số nguyên $m\in B$ thỏa $|a-m|\leq 1000$.

Trở lại bài toán, gọi $a_1<a_2<...<a_{2000}$ là các phần tử của $A$. Khi đó
  • $|a_1-a_{2000}|\geq 1999=2000-1$ nên theo bổ đề 1 ta có $S(a_1)+S(a_{2000})\leq 2017+1=2018$.
  • $|a_2-a_{1999}|\geq 1997=2000-3$ nên theo bổ đề 1 ta có $S(a_2)+S(a_{1999})\leq 2017+3=2020$
  • $|a_3-a_{1998}|\geq 1995=2000-5$ nên theo bổ đề 1 ta có $S(a_3)+S(a_{1998})\leq 2017+5=2022$
  • ............
  • $|a_{992}-a_{1009}|\geq 17=2000-1983$ nên theo bổ đề 1 ta có $S(a_{992})+S(a_{1009})\leq 2017+1983=4000$,
  • $|a_{993}-a_{1008}|\geq 15$ nên theo bổ đề 2 ta có $S(a_{993})+S(a_{1008})\leq 4002$.
  • $|a_{994}-a_{1007}|\geq 13$ nên theo bổ đề 2 ta có $S(a_{994})+S(a_{1007})\leq 4002$.
  • ..........
  • $|a_{1000}-a_{1001}|\geq 1$ nên theo bổ đề 2 ta có $S(a_{1000})+S(a_{1001})\leq 4002$.
Từ đây suy ra $K= S(a_1)+S(a_2)+...+S(a_{2000})\leq (2018+2020+2022+...+4000)+8\times 4002=3016944$.
[RIGHT][I][B]Nguồn: MathScope.ORG[/B][/I][/RIGHT]
 
tikita is offline   Trả Lời Với Trích Dẫn
The Following User Says Thank You to tikita For This Useful Post:
lenguyencm (26-03-2016)
Old 25-03-2016, 12:24 PM   #8
Nguyen Duy Vu
+Thành Viên+
 
Tham gia ngày: Feb 2009
Bài gởi: 6
Thanks: 5
Thanked 5 Times in 2 Posts
Bài hình câu a là trường hợp đặc biệt của bài toán sau: Cho đường tròn (O) cố định, tam giác ABC có B, C cố định và A chạy trên (O); AD, BE, CF đồng quy (D, E, F lần lượt thuộc BC, CA, AB), EF cắt BC tại K, đường tròn đường kính DK cắt (O) tại G, G'. GE, GF cắt (O) lần lượt tại M, N; BM, CN cắt nhau tại H. Tương tự có H'. Chứng minh AH và AH' luôn qua điểm cố định (là trung điểm cung BC) khi A chạy trên (O). Câu b đoạn chứng minh AX, KL, EF đồng quy trong lời giải của bạn Ng. Văn Linh có thể gọi U = AA giao KL rồi dùng Pascal chứng minh được U, E, F thẳng hàng.
[RIGHT][I][B]Nguồn: MathScope.ORG[/B][/I][/RIGHT]
 
Nguyen Duy Vu is offline   Trả Lời Với Trích Dẫn
Old 25-03-2016, 02:04 PM   #9
Nguyen Van Linh
Moderator
 
Tham gia ngày: Aug 2009
Đến từ: Hà Nội
Bài gởi: 277
Thanks: 69
Thanked 323 Times in 145 Posts
Ngày 2.

Bài 4. Cho tam giác $ABC$ nhọn có $\angle ACB<\angle ABC<\angle ACB+\dfrac{\angle BAC}{2}$. Lấy điểm $D$ thuộc cạnh $BC$ sao cho $\angle ADC=\angle ACB+\dfrac{\angle BAC}{2}$. Tiếp tuyến với đường tròn ngoại tiếp tam giác $ABC$ tại $A$ cắt $BC$ tại $E$. Phân giác $\angle AEB$ cắt $AD$ và cắt $(ADE)$ tại $G$ và $ F$, $DF$ giao $AE$ tại $H.$
a) Chứng minh rằng các đường tròn đường kính $AE,DF,GH$ có một điểm chung.
b) Trên phân giác ngoài $\angle BAC $ và trên tia $AC$ lần lượt lấy các điểm $K$ và $M$ sao cho $KB=KD=KM$, trên phân giác ngoài $\angle BAC$ và trên tia $AB$ lần lượt lấy các điểm $L$ và $N$ sao cho $LC=LD=LN.$ Đường tròn đi qua $M,N$ và trung điểm $I$ của $BC$ cắt $BC$ tại $P$ ($P\neq I$). Chứng minh rằng $BM,CN,AP$ đồng quy.

Bài 5. Cho $a_1,a_2,...,a_{n-1},a_n$ ($n\geq 3$), trong đó mỗi số $a_i $ nhận giá trị $\in \{0;1\}$. Xét $n$ bộ số $S_1=(a_1,a_2,...,a_{n-1},a_n)$, $S_2=(a_2,a_3,...,a_n,a_1)$;...;$S_n=(a_n, a_1, ..., a_{n-2},a_{n-1})$. Với mỗi bộ số $r=(b_1,b_2,...,b_n)$, đặt $\omega(r)=b_1.2^{n-1}+b_2.2^{n-2}+...+b_n.2^0.$ Giả sử các số $\omega(S_1); \omega(S_2);...;\omega(S_n)$ nhận đúng $k $ giá trị phân biệt.
a) Chứng minh rằng $n\vdots k$ và $\omega(S_i)\vdots \dfrac{2^n-1}{2^k-1}$ $\forall i=\overline{1,n}.$
b) Kí hiệu $M$ và $m$ lần lượt là max và min của $\omega(S_1),...,\omega(S_n)$. Chứng minh rằng $M-m\geq \dfrac{(2^n-1)(2^{k-1}-1)}{2^k-1}.$

Bài 6. Cho các số thực phân biệt $\alpha_1,\alpha_2,...,\alpha_{16}$. Với mỗi đa thức hệ số thực $P(x)$; đặt $V(P)=P(\alpha_1)+P(\alpha_2)+...+P(\alpha_{16}).$
Chứng minh rằng tồn tại duy nhất đa thức $Q(x)$ bậc 8 có hệ số $x^8$ bằng $1$ thỏa mãn
i) $V(QP)=0$ với mọi đa thức $P$ có bậc bé hơn $8.$
ii) $Q(x)$ có $8$ nghiệm thực (tính cả bội).
[RIGHT][I][B]Nguồn: MathScope.ORG[/B][/I][/RIGHT]
 

thay đổi nội dung bởi: Nguyen Van Linh, 25-03-2016 lúc 03:20 PM
Nguyen Van Linh is offline   Trả Lời Với Trích Dẫn
The Following 3 Users Say Thank You to Nguyen Van Linh For This Useful Post:
huynhcongbang (25-03-2016), lenguyencm (26-03-2016), pco (25-03-2016)
Old 25-03-2016, 02:38 PM   #10
trungnghia215
+Thành Viên+
 
Tham gia ngày: May 2015
Bài gởi: 6
Thanks: 0
Thanked 2 Times in 2 Posts
Bài 1.
Bổ đề 1. $\text{gcd}(a^{m} - 1, a^{n} - 1) = a^{\text{gcd}(m, n)} - 1$
Chứng minh bổ đề. Gọi $d = \text{gcd}(a^{m} - 1, a^{n} - 1)$
Để ý $a^{\text{gcd}(m, n)} - 1 \mid a^{m} - 1, a^{n} - 1 \implies a^{\text{gcd}(m, n)} - 1 \mid d \implies d \ge a^{\text{gcd}(m, n)} - 1$
Mặt khác $a^{m} \equiv 1 \pmod{d}$ và $a^{n} \equiv 1\pmod{p}$ nên $\text{ord}_{d}(a) \mid m, n \implies \text{ord}_{d}(a) \mid \text{gcd}(m, n)$
Hay $a^{\text{gcd}(m, n)} \equiv 1 \pmod{d}$ hay $a^{\text{gcd}(m, n)} - 1 \ge d$.
Kết hợp lại ta có đpcm.
Bổ đề 2. $3^{3^{u}} + 1 = 2^{t}$ có nghiệm duy nhất là $(u, t) = (0, 2)$.
Chứng minh bổ đề. Xét $u \ge 1$, khi đó $3^{3^{u}} = (3^{3})^{3^{u - 1}} \equiv 3^{3^{u - 1}}\pmod{8}$. Đến đây lùi về sẽ thu được $3^{3^{u}} \equiv 3\pmod{8}$. Từ đó ta có $u = 0$ là nghiệm duy nhất.

Giả thiết bài toán quy thành $p$ là ước nguyên tố của $a^{n} - 1$ thì $p\mid d$ với $d = \text{gcd}(a^{n} - 1, a^{3^{2016}} - 1)$
Ta đặt $n = 3^{u}.v$ với $\text{gcd}(v, 3) = 1$.
TH1. $u \le 2016$. Khi đó theo bổ đề $d = a^{3^{u}} - 1$
i) Nếu $v = 1$ thì ta dễ thấy $a^{3^{u}} - 1 \mid a^{3^{2016}} - 1$.
ii) Xét $v \ge 2$, khi đó ta có $a^{v} - 1\mid a^{n} - 1$. Bài toán suy ra nếu $p\mid a^{v} - 1$ thì $p\mid a^{3^{u}} - 1$. Xét $p$ là ước nguyên tố của $\frac{a^{v} - 1}{a - 1} \implies p\mid a^{v} - 1$. Khi đó $p\mid \text{gcd}(a^{v} - 1, a^{3^{u}} - 1) = a - 1$ theo bổ đề trên. Điều này có nghĩa là tập các ước nguyên tố của $a^{v - 1} + a^{v - 2} + \cdots + 1$ phải là tập con của tập các ước nguyên tố của $a - 1$ (*)
Bây giờ ta đặt $A = a^{v - 1} + a^{v - 2} + \cdots + 1 = p_{1}^{d_{1}}p_{2}^{d_{2}}\cdots p_{k}^{d_{k}}$ với $p_{i} < p_{i + 1}$ là các ước nguyên tố.
Theo (*) thì $a - 1 = p_{1}^{c_{1}}p_{2}^{c_{2}}\cdots p_{k}^{c_{k}}.P$
a) Nếu $a$ chẵn thì toàn bộ $p_{i}, P$ lẻ. Áp dụng bổ đề LTE, ta có $v_{p_{i}}(A) = v_{p_{i}}(a^{v} - 1) - v_{p_{i}}(a - 1) = v_{p_{i}}(v)$. Nghĩa là $v = p_{1}^{d_{1}}p_{2}^{d_{2}}\cdots p_{k}^{d_{k}}Q$
Lúc này, $A \ge a^{v - 1} + 1 = a^{p_{1}^{d_{1}}p_{2}^{d_{2}}\cdots p_{k}^{d_{k}}Q - 1} + 1 \ge 3^{p_{1}^{d_{1}}p_{2}^{d_{2}}\cdots p_{k}^{d_{k}}Q - 1} + 1 > A$. Vô lí.
b) Nếu $a = 4t + 1$. Lúc này ta vẫn có thể áp dụng bổ đề LTE như trên do $4\mid a - 1$ nên $v_{2}(a^{v} - 1) = v_{2}(a - 1) + v_{2}(v)$. Đánh giá tương tự ta có điều vô lý.
c) Nếu $a = 4t + 3$. Khi đó để ý nếu $v$ lẻ ta sẽ dẫn đến $v_{2}(A) = v_{2}(v) = 0$ nên đánh giá tương tự a) cho điều vô lý.
Xét $v$ chẵn, theo bổ đề LTE $v_{2}(a^{v} - 1) = v_{2}(a - 1) + v_{2}(a + 1) + v_{2}(v) - 1 = v_{2}(a + 1) + v_{2}(v)$.
Đặt $L = v_{2}(v)$, $K = v_{2}(A) = d_{1}$ và $J = v_{2}(a + 1) \; (J \ge 2)$. Ta có $K = v_{2}(A) = v_{2}(a^{v} - 1) - v_{2}(a - 1) = L + J - 1$ (lưu ý là do ta đã sắp xếp $p_{i}$ nên $p_{1} = 2$)
Viết lại $A = 2^{L + J - 1}p_{2}^{d_{2}}\cdots p_{k}^{d_{k}}$ và $v = 2^{L}p_{2}^{d_{2}}\cdots p_{k}^{d_{k}}.Q$. Mặt khác, để ý là $v_{2}(a + 1) = J$ nên $a + 1 \ge 2^{J} \iff a \ge 2^{J} - 1$. Ta có:
$$A = a^{v - 1} + a^{v - 2} + \cdots + 1 \ge a^{v - 1} + 1 \ge (2^{J} - 1)^{v - 1} + 1 = [(2^{J} - 2) + 1]^{2^{L}p_{2}^{d_{2}}\cdots p_{k}^{d_{k}}Q} + 1 \ge 2^{L}p_{2}^{d_{2}}\cdots p_{k}^{d_{k}}Q(2^{J} - 2) + 2 $$
$$\ge 2^{L + J}.p_{2}^{d_{2}}\cdots p_{k}^{d_{k}} - 2^{L + 1}p_{2}^{d_{2}}\cdots p_{k}^{d_{k}} + 2 \ge A$$
Để ý dấu đẳng thức xảy ra khi và chỉ khi $v = 2$, $a = 2^{L} - 1$.
Từ đó ta cần đi tìm $u$ sao cho mọi ước nguyên tố của $a^{2.3^{u}} - 1$ là ước nguyên tố của $a^{3^{2016}} - 1$. Xét trường hợp $u \neq 0$, theo bổ đề 3 thì $a^{3^{u}} + 1$ có ước nguyên tố lẻ. Xét $q$ là ước nguyên tố lẻ của $a^{3^{u}} + 1$. Ta có $q\mid \frac{a^{3^{2016}} - 1}{a^{3^{u}} - 1}$, nếu ta đặt $x = a^{3^{u}}$ và $y = 3^{2016 - u}$ ($y$ lẻ) thì viết lại $q\mid \frac{x^{y} - 1}{x - 1} = x^{y - 1} + x^{y - 2} + \cdots + 1 = x^{y - 2}(x + 1) + x^{y - 4}(x + 1) + \cdots + x(x + 1) + 1$ (do $y$ lẻ) từ đó suy ra $q\mid 1$ do $q\mid x + 1$. Vô lí.
Từ đó thu được $u = 0$ hay $n = 2$.
Vậy nghiệm là $(a, n) = (2^{L} - 1, 2)$.
TH2. $n > 2016$
Xét tương tự trên, với $v > 1$ ta cũng sẽ thu được $n = 2$. Vô lí.
Xét $v = 1$. Khi đó ta cần chứng minh có 1 ước nguyên tố là ước của $3^{3^{u}} - 1$ nhưng không là ước của $3^{3^{2016}} - 1$
Tuy nhiên điều này không đúng vì nếu ngược lại, nghĩa là sẽ có một ước nguyên tố $q$ sao cho $v_{q}(3^{3^{u}}) > v_{q}(3^{3^{2016}})$. Nếu $q$ lẻ: mọi ước nguyên tố $q$ của $3^{3^{u}} - 1$ và $3^{3^{2016}} - 1$ đều khác $3$ nên theo bổ đề LTE $v_{q}(3^{3^{u}} - 1) = v_{q}(3^{3^{2016}} - 1) + v_{q}(3^{u - 2016}) = v_{q}(3^{3^{2016}})$
Do đó ta chỉ cần xét $3^{3^{u}} - 1 = 2^{i}.(3^{3^{2016}} - 1)$, tuy nhiên PT này vô nghiệm do đó ta có đpcm (có thể cm bằng cách chia qua) :-)
[RIGHT][I][B]Nguồn: MathScope.ORG[/B][/I][/RIGHT]
 

thay đổi nội dung bởi: trungnghia215, 26-03-2016 lúc 12:29 PM
trungnghia215 is offline   Trả Lời Với Trích Dẫn
The Following User Says Thank You to trungnghia215 For This Useful Post:
lenguyencm (26-03-2016)
Old 25-03-2016, 02:58 PM   #11
tikita
Administrator

 
Tham gia ngày: Jun 2012
Bài gởi: 157
Thanks: 2
Thanked 84 Times in 53 Posts
Trích:
Nguyên văn bởi Nguyen Van Linh View Post
Ngày 2.

Bài 6. Cho các số thực phân biệt $\alpha_1,\alpha_2,...,\alpha_{16}$. Với mỗi đa thức hệ số thực $P(x)$; đặt $V(P)=P(\alpha_1)+P(\alpha_2)+...+P(\alpha_{16}).$
Chứng minh rằng tồn tại duy nhất đa thức $Q(x)$ bậc 8 có hệ số $x^8$ bằng $1$ thỏa mãn
i) $V(QP)=0$ với mọi đa thức $P$ có bậc bé hơn $8.$
ii) $Q(x)$ có $8$ nghiệm thực (tính cả bội).
Không biết đề có vấn đề gì không? Vì nếu lấy $P(x)\equiv c$ với $c$ là một số thực bất kỳ thì $0=V(QP)=Q(P(\alpha_1))+...+Q(P(\alpha_{16}))=16Q( c)$. Hay $Q(c)=0$ với $c$ là số thực bất kỳ, do đó $Q(x)\equiv 0$.
[RIGHT][I][B]Nguồn: MathScope.ORG[/B][/I][/RIGHT]
 
tikita is offline   Trả Lời Với Trích Dẫn
Old 25-03-2016, 03:11 PM   #12
Nguyen Van Linh
Moderator
 
Tham gia ngày: Aug 2009
Đến từ: Hà Nội
Bài gởi: 277
Thanks: 69
Thanked 323 Times in 145 Posts
Bài 4.
a) Một lần nữa chúng ta lại thấy bóng dáng của định lý Brocard.
Bổ đề. Cho tứ giác $ABCD$ nội tiếp $(O)$, $AC$ giao $BD$ tại $P, AB$ giao $CD$ tại $E$, $AD$ giao $BC$ tại $F$. Gọi $M$ là điểm Miquel của tứ giác toàn phần $ABCD.EF.$ Khi đó $O,P,M$ thẳng hàng và $M$ là hình chiếu của $O$ trên $EF.$

Đây là một bổ đề rất quen thuộc xin dành cho bạn đọc tự chứng minh.

Quay lại bài toán.
Ta có $\angle AEB=\angle ABC-\angle ACB$ nên $\angle DAF=\angle DEF=\dfrac{1}{2}(\angle ABC-\angle ACB)=90^\circ-(\angle ACB+\dfrac{1}{2}\angle BAC)=90^\circ-\angle ADC$. Do đó $AF\perp BC $ tại $S.$
Suy ra $ (AE), (DF)$ lần lượt là $(ASE), (DSF).$ Gọi $J$ là giao điểm thứ hai của $(ASE)$ và $(DSF)$. Khi đó $J $ là điểm Miquel của tứ giác toàn phần $AEDF.SH$. Theo bổ đề trên ta có $J\in SH$ và $\angle GJH=90^\circ$. Suy ra $(AE), (DF), (GH)$ đồng quy tại $J.$

b) Gọi $B'$ đối xứng với $B$ qua $AK$ suy ra $B'\in AC$ và $B'\in (K).$ Ta có $\angle BDM=\angle BB'M=\angle BAC/2=\angle CAR$ ($R$ là chân phân giác trong). Suy ra $ADRM$ nội tiếp. Tương tự $N\in (ADR).$
Gọi $X$ là tâm $(ADR)$ (không khó nhận ra $X$ chính là $F$). Do tam giác $ADR$ cân nên $AX\perp BC.$ Ta có $\angle XRA=\angle XAR=\angle RAO,$ suy ra $XR\parallel AO$. Mà $AR$ là phân giác $\angle NAM$ nên $R$ là điểm chính giữa cung $MN$, suy ra $XR\perp MN$. Vậy $MN\perp AO$ hay $MN$ đối song với $BC$ trong tam giác $ABC$. Suy ra tứ giác $BNCM$ nội tiếp.
Gọi $Y$ là giao của $BM$ và $CN$, $Z$ là giao của $MN$ với $BC$. Ta có $ZP.ZI=ZM.ZN=ZB.ZC$, suy ra $(PZBC)=-1$, suy ra $P\in AY.$ Ta có đpcm.
[RIGHT][I][B]Nguồn: MathScope.ORG[/B][/I][/RIGHT]
 
Hình Kèm Theo
Kiểu File : png 112.png (73.7 KB, 49 lần tải)

thay đổi nội dung bởi: Nguyen Van Linh, 25-03-2016 lúc 03:18 PM
Nguyen Van Linh is offline   Trả Lời Với Trích Dẫn
The Following 2 Users Say Thank You to Nguyen Van Linh For This Useful Post:
lenguyencm (26-03-2016), Yuki Maki (25-03-2016)
Old 25-03-2016, 07:59 PM   #13
daovuquang
+Thành Viên+
 
Tham gia ngày: Feb 2012
Đến từ: Hanoi
Bài gởi: 2
Thanks: 0
Thanked 1 Time in 1 Post
Cách khác cho câu b bài 3:
Ta chứng minh $T \in (OS)$. Điều này tương đương với $IO \perp IQ$. Kẻ đường thẳng qua $I$ vuông góc $IO$ cắt $BC$,$KL$ tại $X$,$Y$ và cắt $EF$ tại $Q'$.
Kéo dài $EF$ cắt $(O)$ tại $M,N$. $MI,NI$ cắt lại $(O)$ ở $Z,U$. $IY$ cắt $ZU$ tại $R$.

Áp dụng định lí con bướm cho 2 dây $KL$,$BC$, ta có: $IX=IY$.
Áp dụng định lí con bướm cho 2 dây $MN$,$ZT$, ta có: $IQ'=IR$.
Nhận thấy tam giác $IPY$ vuông tại $P$ nên để c/m $Q'$ trùng $Q$, ta chỉ cần chứng minh $Q'P=Q'I$ hay $Q'$ là trung điểm $IY$. Điều này lại tương đương với $R$ là trung điểm $IX$.

Gọi $V,W$ lần lượt là trung điểm $IB$,$IC$. Ta chứng minh $Z,U,V,W$ thẳng hàng.
Lấy $I_b,I_c$ là tâm bàng tiếp góc $B,C$ của tam giác $ABC$.

Do $AICI_b$ nội tiếp nên $EI.EI_b=EA.EC=EM.EN$, hay $I,M,N,I_b$ đồng viên. Tương tự có $I_c,N,I,M,I_b$ đồng viên.
Lại có: $IM.IZ=IB.IK=IV.II_b$ nên $Z,V,M,I_b$ đồng viên.
Tương tự, $IV.II_b=IB.IK=IC.IL=IW.II_c$ nên $V,W,I_b,I_c$ đồng viên.

Từ đây ta nhận thấy: $(VZ,VI_b) \equiv (MZ,MI_b) \equiv (MI,MI_b) \equiv (I_cI,I_cI_b) \equiv (I_cW, I_cI_b) \equiv (VW, VI_b)$,
nên $Z,V,W$ thẳng hàng. Tương tự $U,V,W$ thẳng hàng. Ta có đpcm.
[RIGHT][I][B]Nguồn: MathScope.ORG[/B][/I][/RIGHT]
 
Hình Kèm Theo
Kiểu File : png VN TST 2016 P3.png (55.8 KB, 26 lần tải)
daovuquang is offline   Trả Lời Với Trích Dẫn
The Following User Says Thank You to daovuquang For This Useful Post:
Nguyen Van Linh (26-03-2016)
Old 26-03-2016, 01:47 PM   #14
Nguyen Van Linh
Moderator
 
Tham gia ngày: Aug 2009
Đến từ: Hà Nội
Bài gởi: 277
Thanks: 69
Thanked 323 Times in 145 Posts
Một số mở rộng 2 bài hình:
3b. Cho tam giác $ABC$ nội tiếp $(O)$. $(X)$ là một đường tròn bất kì qua B,C. Phân giác góc $A$ cắt $(X)$ tại $I.$ $BI,CI$ cắt $AC, AB$ tại $E,F.$ $P$ là điểm chia $AI$ theo tỉ lệ k không đổi. Trung trực $PI$ cắt $EF$ tại $Q$. $J$ thuộc $(X) $ sao cho $IJ$ vuông góc $IQ$. Khi đó trung điểm $IJ$ thuộc 1 đường tròn cố định.

$E,F$ có thể xác định bằng cách khác: gọi $(O_b), (O_c)$ là đường tròn ngoại tiếp các tam giác $AIC, AIB$. $IO_b$ cắt $AC$ tại $E$, $IO_c$ cắt $AB$ tại $F$. Kết quả thu được tương tự.

4b. Cho tứ giác $ABCD$. $P$ là điểm nằm trên $AB$ sao cho $\dfrac{AP}{PB}=\dfrac{AD}{BC}$. Đường tròn $(A,AD)$ cắt $PC $ tại $E$, $(B,BC) $ cắt $PD$ tại $F$. Khi đó $B,C,E,F$ đồng viên.
[RIGHT][I][B]Nguồn: MathScope.ORG[/B][/I][/RIGHT]
 
Nguyen Van Linh is offline   Trả Lời Với Trích Dẫn
Old 26-03-2016, 02:25 PM   #15
thaygiaocht
+Thành Viên+
 
thaygiaocht's Avatar
 
Tham gia ngày: Aug 2012
Đến từ: Chuyên Hà Tĩnh
Bài gởi: 165
Thanks: 793
Thanked 216 Times in 93 Posts
Trích:
Bài 4. Cho tam giác $ABC$ nhọn có $\angle ACB<\angle ABC<\angle ACB+\dfrac{\angle BAC}{2}$. Lấy điểm $D$ thuộc cạnh $BC$ sao cho $\angle ADC=\angle ACB+\dfrac{\angle BAC}{2}$. Tiếp tuyến với đường tròn ngoại tiếp tam giác $ABC$ tại $A$ cắt $BC$ tại $E$. Phân giác $\angle AEB$ cắt $AD$ và cắt $(ADE)$ tại $G$ và $ F$, $DF$ giao $AE$ tại $H.$
a) Chứng minh rằng các đường tròn đường kính $AE,DF,GH$ có một điểm chung.
b) Trên phân giác ngoài $\angle BAC $ và trên tia $AC$ lần lượt lấy các điểm $K$ và $M$ sao cho $KB=KD=KM$, trên phân giác ngoài $\angle BAC$ và trên tia $AB$ lần lượt lấy các điểm $L$ và $N$ sao cho $LC=LD=LN.$ Đường tròn đi qua $M,N$ và trung điểm $I$ của $BC$ cắt $BC$ tại $P$ ($P\neq I$). Chứng minh rằng $BM,CN,AP$ đồng quy.
Ý b.


Sử dụng kỹ thuật gọi điểm phẩy (cụ thể là gọi P') kết hợp hệ thức Maclaurin (kiểu VMO 2009) ta thấy mấu chốt là cần chứng minh tứ giác $BNMC$ nội tiếp.

Dùng phép chiếu song song sẽ thấy

$\dfrac{AK}{AL}=\dfrac{AB}{AC}.$

Suy ra tam giác $ABK$ đồng dạng tam giác $ACL$ ($g-c-g$), chú ý $\widehat{KAM}=\widehat{LAN}$ theo tính chất phân giác trong.

Đến đây, ta cần chứng minh được tam giác $AMK$ đồng dạng tam giác $ANL.$

Điều đó đúng do các góc $\widehat{MAL}, \widehat{NAK}$ nhọn và theo Định lý sin thì

$\dfrac{\sin \widehat{AMK}}{\sin \widehat{MAL}}=\dfrac{AK}{KM}=\dfrac{AK}{KB}= \dfrac{AL}{LC} =\dfrac{AL}{LN}=\dfrac{\sin \widehat{ANL}}{\sin \widehat{NAK}}.$

Câu hình a thì chỉ là một tính chất xung quanh Định lý Brocard.


Gọi $AF \cap BC = H'$ thì dùng cộng góc thông thường được $AH' \bot BC.$

Gọi $Q$ là điểm Miquel của tứ giác toàn phần trong hình thì theo trên ta có $Q$ là giao của 2 đường tròn đường kính $AE$ và $BF$.

Theo tính chất của điểm MiquelĐịnh lý Brocard thì $O',G,Q$ thẳng hàng và $O'G \bot HH'.$

Suy ra điểm $Q$ thuộc đường tròn đường kính $GH.$

PS: Điểm Miquel thường là điểm đồng quy của các đường tròn cần chứng minh đồng quy và nó cũng thường là điểm tiếp xúc của các đường tròn cần chứng minh tiếp xúc.


Trích:
Bài 3. Cho tam giác $ABC$ nội tiếp đường tròn $(O)$ có $B,C$ cố định, $A$ chuyển động trên cung $BC$ của $ (O)$. Các phân giác $AD,BE,CF$ giao nhau tại $I$. Đường tròn qua $D$ tiếp xúc với $OA$ tại $A$ cắt $(O)$ tại $G$. $GE,GF$ giao $(O)$ lần thứ hai tại $M,N$. $BM$ giao $CN$ tại $H.$
a) Chứng minh rằng $AH$ đi qua một điểm cố định.
b) $BE, CF$ giao $(O)$ lần lượt tại $K,L$. $AH$ giao $KL$ tại $P$. $Q$ là một điểm trên $EF$ sao cho $QP=QI.$ $J$ là điểm nằm trên $(BIC)$ sao cho $IJ\perp IQ$. Chứng minh rằng trung điểm $IJ$ chuyển động trên một đường tròn cố định.
Ý 3.a có thể dùng tư tưởng phép chiếu xuyên tâm giải kiểu gọi điểm phẩy như sau:



Rõ ràng $GABC$ là tứ giác điều hòa (mô hình này từng xuất hiện trong VMO 2014 và VMO 2010).

Đặt $EF \cap AD=H'; BH' \cap (O)=M', ME \cap (O)=G'.$

Ta chỉ cần chứng minh

$\dfrac{GB}{GC}=\dfrac{c}{b}.$

Thật vậy, ta có

$\dfrac{H'E}{H'F}=\dfrac{AE}{AF}=\dfrac{a+b}{a+c}. $

Suy ra (theo Định lý sin)

$\dfrac{M'E_1}{M'A}=\dfrac{\sin \widehat{H'BE}}{\sin \widehat{H'BF}}=\dfrac{H'E}{H'F}.\dfrac{BF}{BE}= \dfrac{ca}{l_b (c+a)}.$

Do vậy (theo Định lý sin)

$\dfrac{GB}{GC}=\dfrac{GB}{ME_1}.\dfrac{ME_1}{MA}. \dfrac{MA}{GC}=\dfrac{l_b}{ME}.\dfrac{ca}{(c+a) l_b}.\dfrac{ME.ab}{(c+a)}=\dfrac{c}{b}.$


Câu hình 3b sau khi vẽ hình chắc tay để phát hiện ra $IJ$ đi qua $O,$ ta chuyển bài toán về ít điểm như sau:

Cho tam giác $ABC$ nội tiếp $(O),$ ngoại tiếp $(I),$ phân giác trong $BE, CF;$ $D_2$ là trung điểm cung lớn $BC.$ Đường thẳng qua $O$ vuống góc $OI$ cắt $EF$ và $AD_2$ tại $Q$ và $X.$ Chứng minh $IX=4IQ.$



Bài toán này giải qua 2 bước như sau:

Bước 1. Gọi $IQ \cap BC=X_1$ và $X'$ là trung điểm của $AD_2.$

Ta sẽ chứng minh $QX=QX_1.$




Thật vậy, theo một tính chất quen thuộc thì $EF \bot I_aO.$

Theo tiêu chuẩn song song trung điểm thì $O(I_aID_1X')=-1.$

Theo tính chất chùm vuông góc, suy ra $Y(IxBA)=-1$ với $Yx || X'X_1.$

Suy ra $QX=QX_1.$

Bước 2. Chứng minh $IX_1=2IQ.$

Điều này dựa vào bước 1, cụ thể như sau:

Do $Y(X_1QIX)=-1$ nên $\dfrac{QX}{QI}=\dfrac{X_1X}{X_1I}.$

Đặt $X_1X=6a, IX_1=x$ thì tỷ lên trên được viết lại $\dfrac{3a}{3a-x}=\dfrac{6a}{x}.$

Giải phương trình đại số này được $x=2a$ hay bước 2 được chứng minh.

Tóm lại bài toán được giải quyết.
[RIGHT][I][B]Nguồn: MathScope.ORG[/B][/I][/RIGHT]
 
__________________
https://www.facebook.com/thaygiaocht

thay đổi nội dung bởi: thaygiaocht, 26-03-2016 lúc 10:58 PM
thaygiaocht is offline   Trả Lời Với Trích Dẫn
The Following 2 Users Say Thank You to thaygiaocht For This Useful Post:
lenguyencm (26-03-2016), Nguyen Van Linh (27-03-2016)
Trả lời Gởi Ðề Tài Mới

Bookmarks

Ðiều Chỉnh
Xếp Bài

Quuyền Hạn Của Bạn
You may not post new threads
You may not post replies
You may not post attachments
You may not edit your posts

BB code is Mở
Smilies đang Mở
[IMG] đang Mở
HTML đang Tắt

Chuyển đến


Múi giờ GMT. Hiện tại là 07:51 PM.


Powered by: vBulletin Copyright ©2000-2024, Jelsoft Enterprises Ltd.
Inactive Reminders By mathscope.org
[page compression: 126.55 k/142.95 k (11.47%)]